Genetic Counseling Boards Review Questions and Answers (summer 2023/24).

LMNA G608G
A young patient (5yrs) presents for the first time to a new dentist with crowded teeth, making them very hard to clean. In further exploration of the issue, she learns that the child had delayed eruption of his teeth. The patient’s mother also relays that her pediatrician has several times noted how prominent the veins of his scalp are and his short stature. However, she thinks that this is just part of the spectrum seen in their family given that she herself is quite short and has fair skin, making it easier to see her veins. You speak with the dentist who isn’t sure whether or not to refer the family. You are most concerned about what mutation:
LMNA G1138A
LMNA G608G
FGFR3 Exon 11 deletion
FGFR3 G608G

ii, iii
The current prenatal screening test used in your clinic has a sensitivity of 96% and a specificity of 89%. In speaking to representatives from the lab, you learn that they want to adjust their reading of this test by increasing the cutoff levels. Which of the following effects would you expect to see if this change was implemented?
i. Increase sensitivity
ii. Increase specificity
iii. Decrease false positives
iv. Decrease false negatives

None of the above are reasons that MTHFR testing should be conducted
Which of the following scenarios would MTHFR testing be appropriate?
a. Recurrent pregnancy loss
b. Thrombophilia, in the presence of negative primary thrombophilia testing
c. Open neural tube defect identified on prenatal ultrasound, despite adequate folic acid intake
d. Patients about to start a methotrexate chemotherapy regimen
e. none of the above

c
Genetic drift is an exception of which of the following assumptions of Hardy Weinberg:
a. No mutation
b. No migration
c. Population is infinitely large
d. Random mating

Marfan Syndrome (utility of testing influenced by physical evaluation)
For which indication would telehealth service delivery model be inappropriate?
Breast cancer
Marfan Syndrome
Carrier screening
Cascade testing

1/100
–With incidence of disease in AJ being 1/1000, man’s carrier risk is about 1/18
–Woman’s risk of being a carrier is 2/3 since her parents are obligate carriers and she is not affected
— ⅔ x ½ x 1/18 x ½ = 1/108 = ~1/100
A couple is seen for preconception counseling. The woman’s brother has Gaucher disease, and she has Ashkenazi Jewish ancestry (Gaucher disease incidence of 1 in 1,000). Before doing any genetic testing, which of the following is closest to the risk of the couple having an affected child?
1/50
1/100
1/150
1/200

c. Missense mutation in EDA1 *The condition being described is hypohidrotic ectodermal dysplasia which is most often caused by EDA1 mutations.
You are meeting a 3 year old little boy in the pediatric genetics clinic. His parents were referred to the clinic by his astute dentist who mentioned he did not have as many teeth as expected. On meeting the boy, you also note he has sparse hair. His parents also tell you he tends to get overheated when he plays outside. Based on these findings, what is the most likely genetic cause of his signs and symptoms?
a. Missense mutation in IKBKG
b. Deletion in OCA2
c. Missense mutation in EDA1
*The condition being described is hypohidrotic ectodermal dysplasia which is most often caused by EDA1 mutations.
d. Translocation interrupting SPRED1

b. Countertransference *The GC is displaying countertransference as she is transferring her feelings and experiences with RP in her friend onto the client/the family.
A genetic counselor in the ophthalmology clinic is meeting with a family who have a 12 year old son recently diagnosed with retinitis pigmentosa. Throughout the appointment, the patient’s father makes disparaging remarks about blindness and his son being handicapped. The genetic counselor finds herself to be annoyed and angry with the father. The genetic counselor has a good friend with RP and she can’t help but think about him as she talks with the family. What best describes the behavior and feelings of the genetic counselor?
a. Transference
b. Countertransference
c. Resistance
d. Resentment

c. 12.5%
Alport syndrome can be XLR, AD or AR. However, the most common form and the form consistent with the pedigree is XLR. This means for the woman’s mom, there is a 50% chance she is a carrier. She has a 50% chance of passing on that X to the patient. The patient has a 50% chance of passing that onto the future child. Note, that because it’s X linked, females with the condition may or may not manifest in symptoms.
½ ½ ½ = ⅛ or 12.5%
You are meeting with a 30 year old woman to discuss a family history of reported Alport syndrome. She is hoping to start a family in the near future and wishes to know her risk to have a child with the condition. After taking the family history, you notice that only males are affected and there is no male-male transmission. The patient’s maternal uncle is affected. Based on this family history, what is the highest recurrence risk you would quote for this patient to have a child with Alport syndrome?
a. 50%
b. 25%
c. 12.5%
d. Less than 1%

b. Lisch nodules. Learning disabilities and ADHD can be seen in Legius and NF1, lisch nodules are the most common clinic sign of NF1 and do not occur in NF2 or Legius, polydactyly is a key feature of any of these conditions
A 12 year old female is referred to genetics after her PCP finds 5 café au lait spots on physical exam. She also has freckling in her armpits. Which of the following clinical findings would be most helpful for diagnosing her?

a. Learning disability
b. Lisch nodules
c. ADHD
d. Polydactyly

c. If she has ocular telangiectasias. Ocular telangiectasias and ataxia are features of AT and help distinguish this condition from ataxia with oculomotor apraxia
A 15 year old female is referred to genetics for ataxia. Which of the following clinical findings would be most helpful for diagnosing her?
a. Oculomotor apraxia
b. If she currently uses a wheelchair
c. If she has ocular telangiectasias
d. When her ataxia began

d. Germline mosaicism. Rashes that begin as blisters, change to appear wart like and end in hypo or hyperpigmentation that follows blaschko’s lines are characteristic of incontinentia pigmenti syndrome. This condition has XLD inheritance BUT this condition is lethal in males in the first trimester. The only way for dad to pass this on is if he is germline mosaic.
A 1 year old girl is referred to pediatrics clinic due to a skin rash that began as many blisters and slowly changed. It not appears as hypopigmentation that follows the lines of blaschko. After taking the family history you find that she has an 11 year old paternal half sister who presented with the same symptoms as an infant. She currently has hypo and hyperpigmented skin, problems with her eyes and a learning disability. What is the most likely pattern of inheritance in this family?
a. Autosomal Recessive
b. X linked dominant
c. X linked recessive
d. Germline mosaicism

a. All of their sons will be unaffected. Firstly, mutations in COL4A5 cause XL-Alport syndrome (COL4A3 and COL4A4 associated with AD/AR). Anterior lenticonus =Alport. Both hemizygous males and heterozygous females with XLAS are at risk for ESRD.
You are seeing a 25y pregnant patient whose partner has renal disease, anterior lenticonus, and a COL4A5 pathogenic variant. They want to know about the risks to their children. Which of the following is a true statement:
a. All of their sons will be unaffected
b. All of their daughters will be unaffected carriers
c. Each of their children has a ½ chance of inheriting the mutation
d. Both A and B

c. Males with Fabry can reproduce (and also live long enough to reach reproductive years). Fabry is usually inherited.
In certain X-linked conditions, 1/3 of isolated males (i.e., no family history) are de novo, while 2/3 have mothers who are unaffected carriers. Which of the following conditions does this NOT apply to?
DMD
Lesch-Nyhan syndrome
Fabry
Menkes

A. Calculate the coefficient of relationship (r) and the coefficient of inbreeding (F). r = ½ ^4 (same as for first cousins once removed). F = r X ½ .
You are seeing a couple for preconception counseling and the family history is remarkable for consanguinity. They are aware of the potential increased risks for autosomal recessive conditions and birth defects and have done additional reading on the topic to prepare their questions for this visit. They want to know (a) how closely they are related and (b) the probability that their offspring will inherit two copies of an allele that are the same (IBD). They are half-first cousins.
a. 6.25%; 3.125%
b. 3.125%; 1.5625%
c. 12.5%; 6.25%
d. 3.125%; 6.25%

d. 1/2
You are seeing Susie and her partner for a prenatal genetic counseling appointment. Susie and her partner are overall feeling low stress about the pregnancy but want to learn more about their risks because of Susie’s family history of DMD. Susie had an uncle who passed from DMD and has no other maternal aunts or uncles. Susie has three unaffected sons and is currently pregnant with a boy. Susie said her mother never had genetic testing, but Susie did and is a carrier of DMD.
What is the chance that this pregnancy will have DMD?
a. 1/17
b. 1/34
c. 1/68
d. 1/2

b. Hb-Bart Syndrome
You are meeting with a patient who has a family history of a hemoglobinopathy. In gathering the family history, you learn that the patient is a poor historian and communication in the family hasn’t been great. However, the patient tells you that her sister’s baby was affected during pregnancy with ultrasound findings that prompted the initial testing. The most likely diagnosis is:
a. Beta-Thalassemia Major
b. Hb-Bart Syndrome
c. Beta-Thalassemia Minor
d. Sickle Cell Anemia

c. No further testing is needed as this infant does not have SCD
An infant screens positive on NBS for sickle cell anemia. Upon further hemoglobin analysis, she is found to have significant levels of HbS on hemoglobin assay but no detectable levels of an abnormal beta globin chain variant. Based on these tests, which of the following is not a possible next step
a. Molecular sequencing
b. Isoelectric focusing
c. No further testing is needed as this infant does not have SCD
d. No further testing is needed as this finding confirms a diagnosis of SCD clinically

b. Telling your prenatal patient that you are uncomfortable proceeding and refer her on to your colleague, helping to establish the relationship
You previously (3 yrs ago) saw a patient whose father and maternal grandmother had HD. After counseling your patient decided that he did not want to go through with predictive testing. Since switching practice areas in your hospital, you are now scheduled to see a patient this afternoon who is 10 weeks pregnant and would like to discuss with you her husbands family history of HD. You tried to reach her earlier in the week to get more information, but were unsuccessful. When she arrives you notice she is alone and throughout the session you learn, go figure!, that her husband was previously your patient. The mother of the baby is now concerned about the pregnancy and would like to have prenatal testing, citing that her husband has given her written permission to have the test, but can’t take off work for any appointments. Concerned about acting ethically, which of the following proceedings would be best supported by NSGC Code of Ethics:
a. As long as you don’t disclose that you know of her husband, there is no conflict of interest so you may proceed with no additional consults
b. Telling your prenatal patient that you are uncomfortable proceeding and refer her on to your colleague, helping to establish the relationship
c. Order the testing for your prenatal patient given that you no longer has a care-giving relationship with the father of the baby and thus have no responsibility
d. All of the above are viable options

d. Autonomy
Fidelity and veracity are principles that most directly support:
a. Justice and fairness
b. Beneficence
c. Nonmaleficence
d. Autonomy

d. Higher incidence of cis carriers in Asian populations than any other ethnicity. This increases the risk of having a child with Hb bart
Individuals in what ancestral group have the highest risk to have a child with HB Bart?

a. African American
b. Mediterranean
c. Caucasian
d. SE Asian

b. Non maleficence
A 7 year old female patient is referred to genetics clinic due to short stature. You discuss genetic testing with the family, the benefits, limitations and risks. When you describe how they do the test (a blood draw) the patient starts sobbing uncontrollably. The parents tell you that she is terrified of needles and has recently started therapy due to this fear and the psychological impact of having blood draws and shots at the doctors office. You and the family decide together to delay the testing until such a time when she is more emotionally prepared. What ethical principle does this decision represent?
a. Beneficence
b. Non maleficence
c. Autonomy
d. Justice

b.
ACMG firmly recommends against DTC but you could still help the kid interpret the results after, I just can’t envision a scenario where a GC would order a DTC test for the kid.
Incontinentia pigmenti is lethal in males in the first trimester and presents in the first few months of life.
AT causes ataxia early in life, this kid clearly doesn’t have it and there is no changes in screening recommendations for ATM carriers in childhood.
Offer the kid sickle cell testing because it may be important for his health, make sure he has good counseling.
In which of the following scenarios would you offer genetic testing for a healthy 12 year old male child?
a. This child is in foster care and a couple would like to adopt him. The couple has a family history of an ATM mutation and are adopting to help avoid having a child with this type of mutation. They would like to test the child for ataxia telangiectasia and ATM carrier status.
b. This child has a strong family history of sickle cell anemia and his parents are concerned because he is about to start high school basketball
c. The child’s sister has incontinentia pigmenti and his parents are concerned about an unusual rash he got a few months ago.
d. The child read about 23andme online and says he is going to do it with or without you. He asks if you will help him order the test kit and explain the results.

b. Do it for both of them due to time constraints and because ACMG said so in their practice guideline.
A 32 year old woman who is 15 weeks pregnant comes to clinic with her partner for carrier screening. Which of the following testing options do you do?
a. Carrier screening for the women first, then if positive her partner gets testing
b. Carrier screening for the women and her partner simultaneously
c. Do not do carrier screening, just test the pregnancy
d. Only offer screening based on the family history

b. (indicative of alpha thal)
Blood tests for thalassemias are cheaper and faster than genetic testing and often the first test that is done.
People with alpha thal have low MCV (mean corpusal volume) and normal HBA2 and HbF. HbA2 is composed of alpha and delta subunits. It is elevated in beta thal but not alpha thal because in beta thal the delta chain makes hemoglobin because the beta chains are depleted. There is no substitute for alpha chains so HbA2 remains normal.
An infant presents to clinic with anemia, mild jaundice and hepatosplenomegaly. This child did not have newborn screening. What test do you order first and what is the most likely result?
a. Genetic testing-loss of all 4 HBA1 and HBA2 alleles
b. Blood test-low MCV and normal HbA2 and HbF
c. Genetic testing-loss of 3 out of 4 HBA1 and HBA2 alleles
d.Blood test -low MCV and low HbA2 and HbF

c. *Cancer risks are adult-onset
You are meeting with a 38 year old woman for disclosure of a recently identified cancer predisposition syndrome. At the return visit to discuss the results, the woman is deeply concerned about what the results mean for her children, who are now 6, 10, and 12 years old. A pathogenic mutation in which of the following genes would not be recommended to test in her children?
a. MEN1
b. SDHB
c. MUTYH
d. FH

c. *Susan’s autonomy to make decisions about entering the research trial
You are meeting with a 25 year old woman and her parents in the cardio genetics clinic. The patient was seen at an outside institute and had genetic testing related to hypertrophic cardiomyopathy. Unfortunately, genetic testing did not find the cause of her heart condition. Now, the family is coming to you to discuss enrolling in a cardiovascular genetics research study that aims to find new genes for HCM. During the course of the session, you begin to suspect the patient may have a degree of intellectual disability. Once you have taken a family and medical history, you ask the patient, “Susan, when you need to make choices about your health, like starting a new medication, do you make those choices? Or do your parents make those choices with you?” The patient answers that she makes the choices but her parents help her. Her mother helps clarify that they are not medical decision makers for Susan. With that detail, you start to go through the informed consent for the project with Susan and her parents, stopping frequently to gauge Susan’s understanding and answer any questions she or her parents have. What is the primary ethical principle you are upholding in this encounter?
a. Non-maleficence
b. Beneficence
c. Autonomy
d. Justice

d. *Given the ethnicity of the couple are both from higher risk populations for hemoglobinopathies, carrier screening for both is recommended.
You are meeting for a preconception counseling appointment with a couple who are both of Indian descent. The female partner has a sister who passed away shortly after birth of hydrops fetalis. The male partner has no history of thalassemia in his family. The couple is concerned about the possibility of hemoglobinopathies in a future pregnancy. What is the test you will recommend?
a. Blood cell counts on the female partner only
b. Blood cell counts on both partners
c. Blood cell counts and hemoglobin electrophoresis on the female partner first
d. Blood cell counts and hemoglobin electrophoresis on both partners

a. because *The father wants to understand recurrence risk which is influenced by two factors: the mother’s genotype and the penetrance for the condition. While the other statements are true, they do not provide any education or context for the father to understand risks for his daughter.
You are meeting with a 40 year old male patient who was recently diagnosed with HFE related hemochromatosis after experiencing extreme pain and fatigue for about 10 years. He has a daughter who is 16 years old. After learning the condition is inherited, he wants to know what the chances are his daughter will have the condition. What is the next best response to give to this father?
a. “This is difficult to determine without some more information. A key piece of information is if her mother is a carrier for the condition. Perhaps it would be helpful to talk about how this condition is passed in families.”
b. “Even if she has the genes for hemochromatosis, it is unlikely she will get the condition”
c. “This condition is considered adult onset, so we wouldn’t be able to test your daughter now for the condition.”
d. “This condition is usually less severe for females who are menstruating”

A *It is a good first step to acknowledge the provider’s frustration and highlight why you cannot disclose this information. While the other responses may be ways to address the situation (except D, don’t do D), this is a good first statement.
You are a new genetic counselor working in a cancer genetics center and starting to get referrals from oncologists at your institution. One day, a breast cancer oncologist calls you to say she is referring a patient to you “Her sister just saw you and was positive for a breast cancer gene. Can you look up what gene it was in and tell me if it impacts my patient?” The provider tells you the patients name and it matches the last name of a woman who you met two weeks ago for a positive genetic test result. The woman you saw specifically stated she did not want her results shared with anyone and declined signing a release of information document. You call the referring provider back to let him know “I can’t disclose what gene change was seen in her sister.” The referring provider becomes irate and starts to yell at you over the phone. “But you saw the sister! You can at least tell me if this impacts her sister!” What is a good first response to this physician?
a. “I understand you’re upset but I must respect the privacy of patients”
b. “Can I meet with your patient to do genetic testing?”
c. “You patient can try to discuss the results with her sister. I am happy to talk with your patient to help her write a letter to her sister explaining the importance of the information for her sister’s health.”
d. “I suppose there’s no harm in telling you the results of the sister’s genetic test.”

C.
A is wrong because she doesn’t have symptoms of hemachromatosis
B is really invasive and not that helpful
D isn’t technically a bad choice, but is not the best first test because porphyria has many genes that cause abdominal pain/vomiting, but the best first test is to clinically diagnose her with porphyria by looking for the byproduct in her urine
A 25 year old woman has been referred to genetics by her doctor after multiple episodes of unexplained intense abdominal pain followed by vomiting. What would be the best first test based on your differential?
A. Urine analysis for iron
B. Stomach biopsy
C. Urine analysis for porphobilinogen
D. Sequencing of HMBS

d.
Hemoglobin C disease is not a form of sickle cell disease. At least on of the of the biallelic variants needs to be the Hb S allele in order to classify as sickle cell disease
You are seeing a couple for preconception counseling. They mention that in their previous pregnancy, they underwent hemoglobin electrophoresis which revealed they are both Hb C/A. Which of the following is the most correct statement?
a. Their offspring each have a 25% chance for sickle cell disease
b. Their offspring are most likely not at risk for sickle cell disease
c. Their offspring may be at risk for mild anemia
d. Both B and C

b. Engage the mother in an ongoing discussion about the benefits and harms of nondisclosure and her daughter’s interest in the information
A 38-year-old woman is diagnosed with Li-Fraumeni syndrome and the woman has an 11-year-old daughter, Karen. The geneticist recommends that the child be tested for the Li-Fraumeni genetic variant. The mother is concerned about the impact of testing and diagnosis on Karen’s psychological well-being. She describes Karen as “highly strung” and as “a worrier.” The child has been diagnosed with an anxiety disorder and is managed by a psychologist for counseling. The child is otherwise well. The mother requests that testing be done without disclosing it to the child by adding the test onto routine blood work done for another reason and
requests that the results only be revealed if they are positive.
a. Defer testing or discussion until the child is 18 and can legally make decisions for herself
b. Engage the mother in an ongoing discussion about the benefits and harms of nondisclosure and her daughter’s interest in the information
c. Test the child secretly, and only tell her if the result is positive for the genetic syndrome
d. If the mother refuses testing or disclosure, seek a court order

d. Justice
You have recently accepted a position as a genetic counselor for a research project that investigates the underlying etiology of rare genetic disorders. Enrollment in the study requires patients to come to your site 2 times a year for sample collection and relevant tests. You are located in a major metropolitan area. You are currently working on securing funding for travel assistance for participants. The ethical principle you are most concerned about here is:
a. Beneficence
b. Autonomy
c. Veracity
d. Justice

c. Submit an IRB proposal and request a waiver of consent
A genetic counselor is designing a retrospective study to clarify the number of patients referred to connective tissue clinic that had a clinical aortopathy panel ordered and a positive result. The information will be de-identified prior to analysis. Which of the following actions is needed before this information can be queried in the electronic medical record system?
a. Contact each patient and complete informed consent over the phone
b. Provide informed consent at each patient’s next follow-up appointment
c. Submit an IRB proposal and request a waiver of consent
d. Contact Epic support to complete the query and de-identify the data securely

D, Justice
A genetic counselor is working in the Cystic Fibrosis Clinic. The first patient of the day, with homozygous F508del mutations, reports improved symptoms since starting Lumacaftor/Ivacaftor. The genetic counselor is concerned because the second patient of the day, who also has homozygous F508del mutations, is not doing well and shares that he decided not to fill the prescription of Lumacaftor/Ivacaftor because it would be too expensive. The counselor shares that she is going to work with him and the Financial Assistant Team at the hospital to work on coverage for the medication. Under what Code of Ethics principle is the genetic counselor acting?
a. Autonomy
b. Beneficence
c. Nonmaleficence
d. Justice

a. Other clinical intervention already available and the pt is enrolled in, also he is 15 and has a say (assent)
A genetic counselor is the research coordinator of a clinical trial for an oral enzyme replacement product as an alternative to the current standard of care, infusion, in Fabry Disease. A 15 year old boy is diagnosed with Fabry Disease and referred to the genetic counselor to discuss being a part of the clinical trial. The patient and his mother are very engaged during the session and both ask appropriate questions. The patient’s mother is very enthusiastic and shares that she thinks this would be less burdensome then coming to the hospital regularly for infusions. The patient expresses that this feels like more work for him to remember on his own and does not want to do it. What is the BEST action for the genetic counselor to take next?
a. Respect the patient’s desire and do not enroll him in the trial without his assent
b. Respect mom’s decision to provide consent for the patient and enroll him in the trial
c. Consult with the Ethics Review Board to get their input on if you should respect the patient’s or mom’s request
d. Go through the informed consent again to make sure the patient understands the difference between oral and infused enzyme replacement

b. GCs and their colleagues
A 2 day old female with multiple congenital anomalies is admitted to the hospital. Genetics is consulted due to concern for an underlying genetic etiology. No obvious genetic condition is noted and Genetics recommends referring the patient to the research genome study. Before the clinical research coordinator approaches the family, the genetic counselor requests a team meeting with Genetics, Research, and Neonatology to discuss and come to a consensus as a team on what will be best for the patient. Which principle of the NSGC Code of Ethics BEST aligns with the genetic counselor’s action?
a. Genetic counselors themselves
b. Genetic counselors and their clients
c. Genetic counselors and their colleagues
d. Genetic counselors and society

b. An online survey using a Likert scale emailed to patients asking about their experience with family variant group counseling sessions.
Which of the following study designs does not require participants to sign an informed consent document?
a. A prospective trial of MYH mutation positive individuals to evaluate the efficacy of increasing frequency of echocardiograms
b. An online survey using a Likert scale emailed to patients asking about their experience with family variant group counseling sessions.
c. A study to determine the impact of educational outcomes for adult patients with Charcot Marie Tooth who are referred to Genetics after confirmatory molecular testing is completed by Neurology.
d. A study assessing the clinical utility of a neonatal panel compared to whole exome sequencing for infants admitted to the hospital after birth with concern for a genetic condition

b. Angelman syndrome; maternal deletion of 15q12
You are meeting a family in the pediatrics developmental clinic for evaluation of their 3 year old daughter. The daughter’s medical history was noted for a normal birth and first year of life, however, her parents noted she had delayed walking and at 3 years old, she is nonverbal. You notice that when she walks, she looks unbalanced and her legs have a tremor to them. Despite these challenges, her parents report she is incredibly happy and laughs all the time. Based on these findings, which condition and etiology is highest on your differential?
a. Angelman syndrome; paternal deletion of 15q12
b. Angelman syndrome; maternal deletion of 15q12
c. Smith Magenis syndrome, 17p deletion
d. Smith Magenis syndrome, 17p duplication

a. Less than 1%
You are meeting with a couple for preconception counseling. They have a 6 year old son with Prader Willi syndrome. They have waited to have more children because they have been anxious about recurrence risks for Prader Willi. What is the most likely recurrence for Prader Willi?
a. Less than 1%
b. 10-15%
c. 25%
d. 50%

EYA1 (Brachootorenal syndrome)
You are seeing a 23 year old pregnant female in the genetics clinic. The patient has sensorineural hearing loss diagnosed in early childhood. She also has a preauricular pit. She was referred to the genetics clinic after her 20 week ultrasound revealed unilateral renal agenesis in her pregnancy. Mutations in which of the following gene could explain her findings and possibly those of her pregnancy?
TCOF1 (Treacher Collins)
EYA1 (Brachootorenal syndrome)
CHD7 (CHARGE)
COL2A1 (Stickler)

a. Russel Silver syndrome (has characteristic facies and clinodactyly but not associated with polydactyly)
A 2 month old baby girl was referred to the genetics clinic for polydactyly and abnormal facial features. Unfortunately, not much else was noted on the referral to the clinic. Based on these features, which of the following conditions would not be on your differential?
a. Russel Silver syndrome
b. Bardet-Beidl syndrome
c. Greig cephalopolysyndactyly
d. Smith-Lemli-Opitz syndrome

c. Giving advice
You are meeting with a couple to discuss the positive results of their NIPT screening. After describing the results and possible options, the couple is in conflict on whether or not to have an amniocentesis. You tell the couple, “I would encourage you to continue this conversation at home, look at both of your opinions about this, and take some time to reach a decision.” Your statement can be best described as:
a. Non directive counseling
b. Self disclosure
c. Giving advice
d. Directive counseling

a. Kearns Sayre Syndrome (characterized by high CSF level, normal is 15 to 45 mg/dL)
An 18 year old male comes presents in clinic with difficulty moving his eyes and night blindness. Further lab testing indicates that he has a CSF level of 200mg/dl. What is the most likely diagnosis?
a. Kearns Sayre Syndrome
b. Pearson Syndrome
c. Leber’s hereditary optic neuropathy
d. MELAS

b. Ophthalmoplegia, characteristic of MNGIE
A 5 year old patient comes to clinic with failure to thrive, hypotonia and developmental delays. While taking a medical history, which of the following symptoms would make you most suspicious of a mitochondrial disorder?
a. Genitourinary anomalies
b. Ophthalmoplegia
c. Retinitis pigmentosa
d. Elevated oratic acid

d. RAI1 mutation
a = prader-willi
b = bardeyt biedl
A 10 year old patient presents to clinic with obesity. Which of the following is NOT on your differential list?
a. Chromosome 15 maternal uniparental disomy
b. Mutations in the BBS1 gene
c. 1p36 deletion syndrome
d. RAI1 mutation

d. Has she had imaging of her heart or any heart defects
Joubert syndrome doesn’t have heart defects
A 12 year old female presents to clinic with ataxia. After taking the medical history you learn she had hypotonia as an infant. The physical exam reveals oculomotor apraxia and rapid breathing. Which of following would be LEAST important to discuss with the family?
a. Has the patient ever had a brain MRI
b. Genetic testing for a panel of genes related to her symptoms
c. How was her development? Did she walk and talk on time?
d. Has she had imaging of her heart or any heart defects

c. GLI3 frame shift mutation
GLI3 truncation = Pallister-Hall syndrome (only one with hypothalamic hamartoma)
GLI3 point mutation = Greig cephalopolysyndactyly syndrome
FGD1 mutations = Aarskog-Scott syndrome
POR = cytochrome P450 oxidoreductase deficiency
A patient presents to clinic with a hypothalamic hamartoma. Which genetic test results are most likely?
a. GLI3 point mutation
b. FGD1 deletion
c. GLI3 frame shift mutation
d. POR point mutation

II, III, IV
Consanguinity is not really relevant here. SMA + CF carrier screening = universal. 3 or more recurrent miscarriages warrants offering karyotype. To note: from guidelines, genetic testing should not be offered solely based on consanguinity.
You are seeing a couple for preconception counseling who are first cousins. They have a history of three miscarriages. Otherwise, the family history is noncontributory and they are both of Northern European ancestry. You offer the couple:
I. No testing is warranted
II. Karyotype
III. CF carrier screening
IV. SMA carrier screening

CYP21A2 (causes classic CAH, but not involved with sex determination)
SRY = involved in typical male sex development
SF1 (or NR5A1) = involved in gonad determination
MAP3K1 = determines sex characteristics
Mutations in SF1 and MAP3K1 cause swyer syndrome
Which of the following is not a gene that is associated with sexual determination?
SRY
SF1
CYP21A2
MAP3K1

b. 2/3
The chance that trisomy rescue will not result in UPD is
a. 1/3
b. 2/3
c. ¼
d. 0

d. Parental karyotypes will be able to offer more information about the recurrence risk
A newborn’s karyotype results revealed 46, XY,der(14;21)(q10;q10),+21. What can you tell the parents’ about their recurrence risk?
a. There is a 5-15% risk for Down syndrome in future pregnancies
b. There is a 40% chance one of them is a carrier of a balanced translocation
c. Having another child with Down syndrome is rare
d. Parental karyotypes will be able to offer more information about the recurrence risk

a. Heterozygotes for 5T may have male infertility
Not true, homozygotes will have infertility due to congenital absence of vas deferens
Which of the following is NOT TRUE about CFTR 5T allele?
a. Heterozygotes for 5T may have male infertility
b. Compound heterozygotes that have one allele with 5T and one with a severe mutation may have symptoms
c. An allele that has 5T as well as a mild mutation may function as a severe mutation
d. About 10% of the population carries a 5T allele

d. Both A and C
XY female could inherit Y chromosome from dad (w/o SRY). XX male could inherit X chromosome from dad (w/ translocated SRY)
Translocation of SRY to the X chromosome causes:
a. XY females
b. XY males
c. XX males
d. Both A and C

a. general population risk <1%
McCune Albright syndrome is caused by postzygotic somatic mutations in GNAS and is not inherited. Therefore, recurrence risk is not elevated over the general population risk.
You are seeing a couple for prenatal genetic counseling. The couple has a daughter with McCune Albright syndrome. What is the recurrence risk for McCune Albright in their current pregnancy?
a. General population risk, <1%
b. 25%
c. 50%
d. 50% if the pregnancy is male

a. Human cloning by somatic cell nuclear transfer for reproductive purposes
NSGC does not support SCNT for reproductive purposes due to ethical concerns as well as evidence of decreased reproductive success and lifespan in animals who have been conceived with the technology.
Which of the following is not supported by the National Society of Genetic Counselors?
a. Human cloning by somatic cell nuclear transfer for reproductive purposes
b. Stem cells in research and clinical applications
c. Newborn blood spot storage for research and quality control testing
*Note, this is accompanied by the statement that the process should be transparent and voluntary.
d. Return of incidental findings in genetic testing for conditions which have a serious, well-defined medical risk and have implications for clinical management

b. Testes
CAH causes excess corticosteroid production due to mutations in the CYP21A2 gene. As this mutation does not affect the SRY gene, a female with CAH would not have testes but would have varying degrees of virilization due to excess androgen hormones, leading to ambiguous external genitalia and hirsutism.
You are seeing a 15 year old female (46, XY) patient in the DSD clinic a confirmed diagnosis of Congenital Adrenal Hyperplasia. You would expect to see the following signs EXCEPT?
a. Ambiguous external genitalia
b. Testes
c. Uterus
d. Hirsutism

b. Duchenne muscular dystrophy
DMD is due to out of frame deletions or duplications in the dystrophin gene.
All of the following are examples of trinucleotide repeat disorders EXCEPT?
a. Fragile X
b. Duchenne muscular dystrophy
c. Myotonic dystrophy
d. SCA1

c. Kallmann Syndrome
Type 2 is AD (FGFR1), Type 1 is XL-R (KAL)
All of the following are inherited exclusively in an X-linked recessive manner except _.
a. Androgen Insensitivity Syndrome (AIS)
b. Adrenal Hypoplasia Congenita
c. Kallmann Syndrome
d. FANC-B

b. ATM; four
Ataxia telangiectasia is an autosomal recessive condition that is characterized by an increased susceptibility to cancer (especially leukemia and lymphoma). Carriers of mutations in _ also have an increased cancer risk that is _ times the population risk.
a. AR; four
b. ATM; four
c. ATM; two
d. FGFR1; three

a. FANCB; x-linked recessive
Note: FANCD1 = BRCA2
Most forms of Fanconi Anemia are inherited in an autosomal recessive manner, except complementation group which is inherited in a __ manner.
a. FANCB; x-linked recessive
b. FANCA; autosomal dominant
c. FANCD1; x-linked recessive
d. FANCB; autosomal dominant

c. Person-centered Counseling (Rogers, 1992)
Unconditional positive regard, empathy and counselor genuineness are three key tenets of counselor attitudes in which of the following models of genetic counseling
a. Person-oriented Counseling (Kessler, 1979)
b. Family Systems Counseling (Weil, 2000)
c. Person-centered Counseling (Rogers, 1992)
d. Intersystems Counseling (Eunpu, 1997)

D.WGS
Which of the following tests has the lowest average depth coverage and thus the lowest analytical sensitivity?
A. Targeted Panel
B. Single Gene Sequencing
C. WES
D.WGS

B. Reflecting
Which interviewing technique involves repeating the last phrase a client says as a question?
A. Questioning
B. Reflecting
C. Rephrasing
D.Redirecting

B. Male with unilateral cleft lip
Males are more likely to have a cleft lip/palate than females. There is a higher genetic contribution to affected females. A more severe presentation also indicates a higher genetic contribution. The lowest risk is with the male with the least severe presentation.
Which individual has the lowest risk of having a child with a cleft lip and palate?
A. Male with bilateral cleft lip and palate
B. Male with unilateral cleft lip
C. Female with bilateral cleft lip and palate
D. Female with unilateral cleft lip

0.8%
What percent of cases with a negative CMA result would be expected to have a balanced rearrangement that would be detected by a karyotype?
10%
3.5%
0.8%
<0.1%

a. Newborns or infants presenting with neonatal diabetes mellitus.
According to ACMG recommendations, in which of these situations should UPD testing be considered?
a. Newborns or infants presenting with neonatal diabetes mellitus.
b. Infants or children who present with multiple congenital anomalies, developmental delay or mental retardation
c. Family history of consanguinity
d. All of the above.

a. Pneumothorax
Which of the following is NOT a major criteria of Marfan syndrome?
a. Pneumothorax
b. Pectus carinatum
c. Aortic dissection
d. High palate

c. Daughters of fathers with CP; sons of mothers with CP
Recurrence risk for non-syndromic cleft palate (CP) is highest for and lowest for .
a. Sons of fathers with CP; daughters of mothers with CP.
b. Sons of mothers with CP; daughters of fathers with CP
c. Daughters of fathers with CP; sons of mothers with CP
d. Daughters of mothers with CP; sons of fathers with CP.

a. Types II, III
Type 1 is most mild, type 4 is variable, type 5 is moderate
Type 2 and 3 are most severe
Which types of OI are reliably detected on prenatal ultrasound?
a. Types II, III
b. Types II, IV
c. Types III, V
d. Types IV, V

46, XX, del(5)(q33q34)
CMA can detect microdeletion syndromes. However, CMA is not able to detect balanced translocations or inversions
Which of the following chromosome abnormalities could be detected by chromosome microarray?
a. 46, XX, inv(4)(p14q21.3)
b. 46,XX, t(13; 14)(13q14q)
c. 46, XX, del(5)(q33q34)
d. 46, XY, t(3;5)(p21;q31)

c. You take note that the patient becomes emotional during the family history when you ask about her father
Psychological attending includes sensing the feelings and attitudes the patient may have. This is in contrast to physical attending which includes the ways the GC would demonstrate or communicate understanding to the patient.
You are seeing a patient in the adult medical genetics clinic referred for evaluation of a connective tissue disorder. As the genetic counselor, you wish to show strong attending skills throughout the session. Which of the following scenarios demonstrates psychological attending to the patient.
a. You nod your head and verbalize “mm-hmmm” to the patient’s speaking
b. You work to remain relaxed throughout the session
c. You take note that the patient becomes emotional during the family history when you ask about her father
d. You try to avoid distracting behaviors such as looking repeatedly at your watch

b. Chromosome microarray
ACMG recommends a CMA as a first line test for autism spectrum disorder.
You are seeing a 5yo male with a recent diagnosis of autism spectrum disorder. He has no other known health problems or congenital anomalies and has not had any previous genetic testing. His parents are interested in pursuing genetic testing to determine the etiology of his autism spectrum diagnosis. What is the most appropriate test to offer?
a. Karyotype
b. Chromosome microarray
c. Whole exome sequencing
d. Multi-gene panel of autism spectrum disorder genes

A nonsense mutation in a gene where a loss of function is a known mechanism of the disease
ACMG states this level of evidence has the strongest support for pathogenicity. Pay attention to the disease mechanism–it must have a known LOF mechanism for a null (nonsense, etc) variant to be pathogenic
You are reviewing testing results for whole exome sequencing of a patient you saw in the pediatrics genetics clinic. When evaluating possible variants for pathogenicity, which of the following evidence alone would offer the strongest support for a pathogenic variant?
a. A nonsense mutation in a gene where a loss of function is a known mechanism of the disease
b. A de novo mutation confirmed with trio testing with no family history of the disease
c. In vivo functional studies supporting damage to the protein product
d. The variant is within a known mutational hotspot of the gene

d. AD, de novo pathogenic variant
A severe presentation of OI is typically not inherited
A 3yo male with speech delay presents to the pediatric genetics clinic. He began walking at 11mo and has had multiple fractures between now and then, all healing normal. What do you tell the family is the most likely inheritance?
a. AD, pathogenic variant passed down from mother
b. AD, pathogenic variant passed down from father
c. AR, both parents are obligate carriers of a pathogenic variant
d. AD, de novo pathogenic variant

c. Chorionic villus sampling to complete biochemical analysis of type 1 collagen
A 28yo female is referred to prenatal genetics clinic because her husband has a clinical diagnosis of osteogenesis imperfecta. She is 10 weeks pregnant and would like to screen her pregnancy for OI. What plan should you recommend to the patient?
a. COL1A1 and COL1A2 sequencing analysis of father then return to discuss prenatal testing once familial variant has been identified
b. Wait until ~15 weeks to complete amniocentesis for COL1A1 and COL1A2 sequencing analysis of the pregnancy
c. Chorionic villus sampling to complete biochemical analysis of type 1 collagen
d. Serial ultrasounds starting at 18 weeks to look for diagnostic skeletal deformities

b. Regions of homozygosity
A 18 yo female is referred to maternal fetal medicine at 20 weeks gestation due to multiple congenital anomalies identified on ultrasound. Social history is significant for father of the baby being a family member. The patient is adamant about protecting the father of the baby and will not reveal his identity. When discussing proceeding with microarray, what is the most important component to discuss with the patient?
a. Size of deletions/duplications that can be detected
b. Regions of homozygosity
c. Uniparental disomy
d. Mosaicism

c. Microarray with reflex to FMR1
A 4 yo female comes to genetics clinic with her foster mother for follow up of global developmental delays. The genetics team had previously evaluated the patient shortly after birth during in-patient consults due to bicuspid aortic valve and dysmorphic facial features. Until now, she was lost to care and no genetic testing had been ordered. She was also recently seen by Developmental Pediatrics and diagnosed with autism. What is the best genetic testing plan to discuss with the family?
a. FMR1 with reflex to microarray
b. Chromosome Analysis with reflex to microarray
c. Microarray with reflex to FMR1
d. MECP2 with reflex to microarray

d. FBN1 sequencing (Marfan)
COL3A1 is EDS
16yo male comes to genetics clinic with his mother due to presentation of pectus carinatum. His surgical history is significant for pneumothorax repair. He is following Ophthalmology for high myopia. Physical exam is notable for pes planus, reduced elbow extension, and scoliosis. Recent echo report notes dilated aortic root. What is the best genetic test to counsel the family on?
a. Chromosome analysis
b. FBN1 deletion/duplication analysis
c. COL3A1 sequencing
d. FBN1 sequencing

c. Mutations in IRF6 (Van der woude)
FGF = Harsfield syndrome, holoprosencephaly
MSX1 = wolf-hirschorn
Orofacial clefts are often multifactorial in nature and attributing causal factors and risk can be challenging due to their complex inheritance. However, several risk factors have been identified, the most significant factor of which is:
a. Maternal cigarette smoking
b. Mutations in MSX1
c. Mutations in IRF6
d. Mutations in FGF genes

Pupils
Upslanting and downslanting palpebral fissures are measured clinically by drawing a line connecting the __ and determining whether this line intersects the inner and outer eye.
Pupils
Internal canthi
External canthi
Midpoint of the inner sclera

a. RUNX2 sequencing analysis (Cleidocranial dysplasia, stranger things)
FGFR3 (achondraplasia)
TWIST1 (Saethre-Chotzen syndrome, craniosynthstosis)
You are asked to see a infant (10 months) whose pediatrician is concerned about his abnormally large fontanels that have not fused and some dysmorphic feature including tapering fingers and shoulders that appear to face towards the midline. Upon further questioning your team identifies that the patient’s mother had limited access to prenatal care. Which test is most important as your team considers testing options?
a. RUNX2 sequencing analysis
b. RUNX2 del/dup analysis
c. FGFR3 targeted mutation analysis
d. TWIST1 sequencing

c. Pfieffer Syndrome
FGFR3 exhibits phenotypic or clinical heterogeneity in that mutations in FGFR3 can cause all of the following conditions except:
a. Hypochondroplasia
b. Thanatophoric dysplasia
c. Pfieffer Syndrome
d. Muenke Syndrome

d. all of the above
You are seeing a patient in your prenatal clinic who was referred for a family history of Ehlers Danlos Syndrome. The patient is unsure of the type and doesn’t have any testing reports from family members. The patient is adamant that any testing during the pregnancy is unnecessary as testing wouldn’t change the outcome of the pregnancy for her and her husband. You and your team believe it is still important for the patient herself to have a clinical evaluation and consider testing during the pregnancy primarily because of the risk all of the following except:
a. Increased extensibility of skin could lead to increased risk of painful stretch marks
b. Increased joint laxity that may make labor more painful
c. Increased risk of uterine rupture during pregnancy
d. All of the above are significant risks that concern your team

c. Sequencing of KMT2D; if positive, up to a 50% chance of having another affected child (Kabuki Syndrome)
A 3 year old male presents to pediatric genetics clinic for developmental delay. Surgical history is significant for repair of coarctation found on echo to evaluate a heart murmur. He has a g-tube for supplemental feeds due to failure to thrive. Clinical evaluation notes short fifth fingers and persistent fetal fingertip pads, long palpebral fissures, and arched eyebrows. The family is interested in pursuing genetic testing to clarify a diagnosis and inform family planning. You discuss the following:
a. Sequencing of KMT2D (also known as MLL2); if positive, up to 25% chance of having another affected child
b. Deletion/duplication of KMT2D; if positive, up to a 25% chance of having another affected child
c. Sequencing of KMT2D; if positive, up to a 50% chance of having another affected child
d. Deletion/duplication of KMT2D; if positive up to a 50% chance of having another affected child

b. Minimal Encouragers
A 5yo female is seen for follow up in pediatric genetics with her mother to disclose microarray results identifying a deletion of 22q11.2. Her mother states, “There are so many different specialists she needs to see and I am worried I won’t remember who all she needs to follow.” The counselor replies, “So, you’re worried?” Which of the following techniques BEST describes this response?
a. Feeling Reflection
b. Minimal Encouragers
c. Content Reflection
d. Paraphrasing

d. Full gene sequencing of CFTR (because of better clinical sensitivity due to ethnic background)
A 4yo female is referred to pediatric genetics for presentation of poor growth, recurrent upper respiratory infections, and abnormal sweat chloride test. You obtain a family history and family reports that their ethnicity is Hispanic. You recommend the following testing:
a. Targeted ∆F508 testing with reflex to full gene sequencing of CFTR
b. ACOG 23 mutation panel with reflex to full gene sequencing of CFTR
c. Genzyme 97 mutation panel with reflex to full gene sequencing of CFTR
d. Full gene sequencing of CFTR

a. FBN2 gene analysis; echocardiogram (crumpled ear = Beal Syndrome)
A 12yo male is referred to connective tissue clinic by pediatric surgery due to pectus carinatum and kyphosis. His history is significant for gross and fine motor delays. Mother reports that the delays were suspected to be caused by joint stiffness in knees, hips, and thumbs which improved with physical therapy. On physical exam, it is noted that his ears have a “crumpled” appearance. Family history is remarkable for father with similar delays and marfanoid habitus. You recommend the following:
a. FBN2 gene analysis; echocardiogram
b. FBN1 gene analysis; renal ultrasound
c. FBN2 gene analysis; renal ultrasound
d. Microarray

c. Content and Feeling Reflection
A 28yo woman presents to genetics clinic due to difficulty becoming pregnant. She reports irregular menstrual cycles. Family history is remarkable for maternal grandfather with Parkinson-like presentation. FMR1 testing results with a trinucleotide repeat size of 150 confirming that she is a premutation carrier. She expresses, “I thought my irregular periods were just because I ran long distances but you are telling me it is because of this Fragile X condition.” The counselor replies, “You are surprised that this genetic test result is explaining your cycles rather than lifestyle.” Which of the following techniques BEST describes this response?
a. Summarizing
b. Feeling Reflection
c. Content and Feeling Reflection
d. Paraphrasing

b. Loeys-Dietz syndrome (ectopia lentis not observed)
A 10 year old boy with a long and lean body habitus, pectus excavatum, highly arched palate, ectopia lentis, and a family history of mitral valve prolapse is seen in your clinic. Which of the following would not be in your differential?
a. Marfan syndrome
b. Loeys-Dietz syndrome
c. Homocystinuria
d. All of the above should be included in the differential

b. Expanded mutation panel (For individuals of non-Northern European descent, pan-ethnic panels that include additional mutations more commonly identified in minority populations are appropriate to consider.”)
A 36yo African American woman who is 14w1d is seeing you to discuss her options for prenatal testing. The pregnancy has been unremarkable except for some morning sickness. She wants to know as much information as she can. Her family history is positive for a first cousin with autism spectrum disorder. In addition to her testing options for chromosomal abnormalities, you discuss carrier screening. What would be the most appropriate test to order for CF carrier screening?
a. Standard CF panel
b. Expanded mutation panel
c. CFTR sequencing
d. None. Testing is not indicated

b. 25%
A couple is seeing you for preconception counseling. A few years ago, their 10 month old son died from complications of Zellweger syndrome. They want to know their chance of having another affected child.
a. 1-2%
b. 25%
c. 50% if it is a male
d. You do not have enough information to answer this

d. All of the above
You are seeing a 32yo woman who is 15w0d and was identified to be a carrier of a FMR1 intermediate allele through population screening from her referring provider. She has read about the results online and is concerned about the complications associated with premutation carriers and the health of her future child. She wants the pregnancy tested to see if it has expanded into a premutation. You are hesitant to order the testing because
a. The test will not predict the phenotype
b. You are concerned about the future child’s autonomy
c. It is unlikely that it will expand into a full mutation
d. All of the above

d. Rett Syndrome, most affected males do not survive to term
Chad, is a two-year-old boy who has come in for a genetics evaluation. At Chad’s last well child visit, at 15 months of age, the history and physical examination were normal, but he was not talking. Chad still has no expressive language. He appears to hear normally and follows simple commands. His gross and fine motor development appear normal for his age. He is described as shy. Parents note that he is not communicative with other children or adults and consistently avoids eye contact. He is frequently irritable and hyperactive at home. Neonatal history was unremarkable except for hypotonia. Before even completing a physical exam, which of the following would NOT be on your differential?
a. Fragile X Syndrome
b. Prader-Willi Syndrome
c. Angelman Syndrome
d. Rett Syndrome

b. Congenital contractural arachnodactyly (mutations in FBN2, fibrillin 2)
Which of the following genetic conditions is not the result of a collagen defect?
a. Osteogenesis imperfecta, Type II
b. Congenital contractural arachnodactyly
c. Ehlers Danlos, classic type
d. Multiple epiphyseal dysplasia

a. How old were his parents at the time of his conception?
You are meeting with a 10 year old male with intellectual disability and a history of developmental delay who has had a normal karyotype analysis and chromosome microarray. In taking the medical and family history, which of the following is the least important question to ask the parents?
a. How old were his parents at the time of his conception?
b. Were there any prenatal exposures to medications, alcohol, or recreational drugs?
c. Is there any family history of intellectual disability or developmental delay?
d. Were there any complications with his birth or immediately following birth?

d. Williams syndrome (supravalvular aortic stenosis is a key and defining type of heart condition for Williams)
You are meeting with a woman who is 20 weeks pregnant with her second child. The ultrasound technician noted supravalvular aortic stenosis and the patient was referred to genetics for a genetic counseling appointment. What condition will you likely discuss with the pregnant woman?
a. Alagille syndrome
b. Costello syndrome
c. Noonan syndrome
d. Williams syndrome

d. Umbilical hernia (can be seen in 22q but at a much lower frequency than the triad of congenital heart defects, palatal anomalies, and immunodeficiencies)
You are meeting 8 month-old boy and his parents. The boy was recently diagnosed with 22q11.2 deletion syndrome after his pediatrician sent for genetic testing. Which of the following signs is least likely to have prompted the pediatrician to consider 22q11.2 deletion syndrome?
a. Ventricular septal defect
b. Submucosal cleft palate
c. Frequent infections
d. Umbilical hernia

c. If the couple both test negative with the carrier panel, there is a residual risk for cystic fibrosis in their current pregnancy
You are meeting with a couple of Northern-European ancestry who are expecting their first child. You discuss carrier screening for cystic fibrosis using a panel of common mutations. What is the most important information the couple should know about carrier screening for cystic fibrosis?
a. There are over 2000 known mutations in the CFTR gene that cause cystic fibrosis
b. The carrier screening panel for cystic fibrosis has a high detection rate
c. If the couple both test negative with the carrier panel, there is a residual risk for cystic fibrosis in their current pregnancy
d. Being a cystic fibrosis carrier is common for those of Northern-European ancestry

d. Acknowledge the patient’s situation, and refer her for psychological services before proceeding.
A 26-year-old woman comes for genetic counseling for presymptomatic testing for Huntington disease (HD). Her psychiatrist has told the genetic counselor that the woman has an unstable living situation and is showing signs of depression. The woman told the psychiatrist that “she has no one.” The woman is very emotional when she arrives at the counselor’s office and demands HD testing. Which of the following is the BEST way to proceed with the session?

a. Explain being uncomfortable arranging her testing, and refer her to another
genetic counselor.
b. Discuss with the patient the statements that she made, and proceed with
the protocol and testing as requested.
c. Proceed with testing because her depression may be an early manifestation
of HD.
d. Acknowledge the patient’s situation, and refer her for psychological services before proceeding.

A.gastrointestinal bleeds
A 13-year-old girl comes to the genetics clinic because of frequent nosebleeds. When taking the family history, which of the following is MOST important for a genetic counselor to elicit?

A.gastrointestinal bleeds
B. varicose veins
C. hearing loss
D. renal carcinoma

C.Ensure genetic testing for the patient and her husband. (it’s AR, not XL)
A woman who was evaluated and diagnosed 5 years ago with limb girdle muscular dystrophy type 2D comes for genetic counseling with her new husband. The couple is considering starting a family in the near future. Which of the following should the counselor do FIRST?
A. Request a consultation with perinatology and cardiology.
B.Check creatine kinase levels and offer genetic testing for the patient.
C.Ensure genetic testing for the patient and her husband.
D.Refer the couple to a fertility group for preimplantation genetic diagnosis.

B.a woman who has t(14q;21q)
Which of the following individuals has the greatest risk of having a child with Down syndrome?

A. a man who has t(14q;21q)
B.a woman who has t(14q;21q)
C. a woman who is 35 years old
D. a woman who has a child with trisomy 21

d. valproic acid (5% have NTD)
A pregnant woman has an ultrasound that identifies a lumbar meningomyelocele in the fetus. Which of the following exposures is MOST closely associated with this birth defect?

a. tetracycline
b. lithium
c. phenytoin
d. valproic acid

C.germline mosaicism (OI is AD, type II is very penetrant)
Ultrasonography of a 26-year-old woman at 16 weeks gestation shows fetal findings consistent with osteogenesis imperfecta, type II. The woman had a previous fetus affected with the same condition. The woman and her husband are healthy and have no obvious clinical characteristics of osteogenesis imperfecta. Which of the following is the MOST likely explanation for this recurrence?

A. autosomal recessive inheritance
B. de novo mutation
C.germline mosaicism
D.incomplete penetrance

A. Acknowledge the mother’s concerns, but inform her that the counselor is obligated to provide accurate information.
A 15-year-old girl at 21 weeks gestation comes with her mother to the genetic counselor because the fetus has gastroschisis. The patient’s mother insists that the birth defect is incompatible with life and that she should terminate the pregnancy, but the patient says she wants to keep the baby. However, the patient says that she might terminate the pregnancy if the counselor says the chances of the fetus surviving are poor. The patient’s mother becomes agitated when the counselor states that the birth defect is often compatible with life. She accuses the counselor of undermining her authority as the mother. Which of the following is the BEST response by the counselor?

A. Acknowledge the mother’s concerns, but inform her that the counselor is obligated to provide accurate information.
B. Remind them that this is the patient’s decision and ask the mother to leave
the room.
C. Discuss that the mother will make the ultimate decision because the patient
is a minor.
D. Reflect the patient’s ambivalence and offer to reschedule the appointment.

C.1/36
A couple who are first cousins inquire about the chance of having a child affected with a genetic disorder. Their common maternal uncle has galactosemia. The chance that they will have child with galactosemia is

a. 1/8.
b. 1/16.
C.1/36.
D.1/64.

A.Confirmatory DNA testing of her mother is needed.
A 30-year-old woman comes for genetic counseling to discuss her recent presymptomatic testing for Huntington disease (HD), which was negative. Her mother was diagnosed with HD at age 36, and her deceased maternal grandmother was diagnosed with HD at age 40. Neither had DNA testing. Which of the following is the MOST important next step to discuss with the woman?

A.Confirmatory DNA testing of her mother is needed.
B. Her mother and grandmother did not have HD.
C. She could still develop HD because of anticipation.
D. She did not inherit the familial condition.

C.nondirective counseling
A pregnant woman is undecided about pursuing invasive testing. She asks, “If I were your wife, what would you tell me to do?” The counselor replies, “I don’t know what I would do, but I do know what concerns I would think about.” Which of the following techniques BEST describes this response?

A. advice giving
B. directive guidance
C.nondirective counseling
D.nonjudgmental reflection

d. 7-dehydrocholesterol analysis
Which of the following laboratory studies is MOST likely to confirm the diagnosis of Smith-Lemli-Opitz syndrome?

a. 15q11-13 methylation analysis
b. isoelectric focusing of transferrin
c. chromosomal microarray
d. 7-dehydrocholesterol analysis

A. Explain that screening tests are not diagnostic.
A couple whose child had a positive newborn screen for galactosemia is seen for genetic counseling. Follow-up testing showed slightly reduced GALT enzyme activity, gal-1-p and galactitol levels within normal limits, and one copy of the Q188R mutation in the GALT gene. Which of the following is MOST important to discuss with the parents?

A.Explain that screening tests are not diagnostic.
B. Recommend the enzyme studies be repeated.
C. Discuss the management of galactosemia.
D.Describe the purpose of newborn screening

C. immunohistochemical testing
A 45-year-old man with colon cancer comes for genetic counseling. In conjunction with microsatellite instability testing, which of the following is the BEST next step in deciding if the tumor is the result of Lynch syndrome?

A. BRAF mutation testing
B. methylation testing
C. immunohistochemical testing
D.KRAS mutation testing

B.daughter with limb girdle muscular dystrophy.
Discussion of artificial insemination with donor sperm is MOST appropriate for couples who have a

A. daughter with cleft lip and palate.
B.daughter with limb girdle muscular dystrophy.
C. son with Becker muscular dystrophy.
D> son with Kearns-Sayre syndrome.

C. hereditary non-polyposis colorectal cancer

  1. A 43-year-old woman seeks genetic counseling because of a recent diagnosis of endometrial cancer. Her family history includes a maternal grandmother with breast cancer at age 56, father with colorectal cancer at age 47, and one paternal aunt with ovarian cancer at age 49. Counseling regarding testing for which of the following syndromes is MOST appropriate?

A. familial adenomatous polyposis
B. hereditary breast and ovarian cancer
C. hereditary non-polyposis colorectal cancer
D. PTEN hamartoma tumor syndrome

D.Allow the mother time to come to terms with the diagnosis at her own rate.
A 2-year-old child with developmental delay is newly diagnosed as having Rubinstein-Taybi syndrome. The mother continues to keep the child’s medical appointments but tells the genetic counselor that she does not believe the diagnosis. Which of the following is the BEST strategy for the counselor to pursue?

A. Refer the mother to a support group to meet with other families.
B. Emphasize the positive aspects of the diagnosis with the mother.
C. Review the medical findings to convince the mother of the diagnosis.
D.Allow the mother time to come to terms with the diagnosis at her own rate.

C. 1/250.
DNA testing for cystic fibrosis (CF) in a 25-year-old Caucasian woman is negative. The test is able to identify 90% of CF carriers. The chance that the woman is a carrier is closest to

A. 1/50.
B. 1/100.
C. 1/250.
D. 1/500.

A.cat-eye syndrome (inverted duplicated ch22)
velocardiofacial (aka 22q11.2 deletion)
Pallister-Killian syndrome (isochromosome 12p)
Russell-Silver syndrome(imprinting defect on chr 7 or 11)
A 37-year-old woman comes to the clinic for amniocentesis at 16 weeks gestation. Fetal karyotype shows an extra de novo marker chromosome derived from chromosome 22. The fetus may be at risk for which of the following syndromes?

A.cat-eye syndrome
B velocardiofacial syndrome
C Pallister-Killian syndrome
D Russell-Silver syndrome

a. bifid uvula (indicative of loeys-dietz, TGFBR1/2)
A 2-year-old boy is being evaluated in the genetics clinic because of a dilated aortic root. Comprehensive testing of FBN1 showed no abnormalities. Discovery of which of the following features is consistent with the MOST likely diagnosis?

A.bifid uvula
b. microtia
c. horseshoe kidney
d. polydactyly

A.”Her maternal grandmother went through menopause early.”
A 4-year-old girl is referred to the genetics clinic to be evaluated because she may have fragile X syndrome. During the appointment, the patient’s mother provides information on the family history. Which of the following statements about her daughter’s family history is MOST suggestive of fragile X syndrome?

A.”Her maternal grandmother went through menopause early.”
B. “Her paternal grandfather and his brother have tremors.”
C.”Her father completed the 9th grade and then dropped out.”
D.”Her maternal uncle has a son with autism.”

B. maternal symptoms of hyperammonemia
A pregnant woman whose son has ornithine transcarbamylase (OTC) deficiency comes for genetic counseling. A detailed ultrasound at 18 weeks shows a normal female fetus. Which of the following is the MOST important information to collect to assess risk in the current pregnancy?

A. results of her partner’s carrier testing
B. maternal symptoms of hyperammonemia
B. orotic acid and ammonia levels in her son
C. maternal diet history

b. Ask the father about the scar on his lip, explaining that this information is essential for risk assessment.
A couple comes for genetic counseling because of advanced maternal age. The counselor notes that the father has a large vertical scar on his upper lip. While taking the family history, the counselor asks both individuals if either has a history of birth defects, and they deny any such history. Which of the following is the BEST next step for the counselor to take?

a. Inquire again about the history of birth defects on both sides of the family, using cleft lip as an example.
b. Ask the father about the scar on his lip, explaining that this information is essential for risk assessment.
c. Avoid embarrassing the father, and tell the couple that ultrasound can
detect certain birth defects like cleft lip.
d. Tell the perinatologist about the scar, so she can ask the couple again
about a history of birth defects.

d chromosomal microarray.
A 3-year-old boy is referred to the genetics clinic for evaluation of his autistic-like behaviors and cognitive impairment. The test with the highest diagnostic yield is
a. FMR1 analysis.
b. mitochondrial sequencing.
c. urine organic acid analysis.
d chromosomal microarray.

c. karyotype results on the child
A woman is referred for genetic counseling. Her son died at the age of 3 weeks with the following findings: holoprosencephaly, cleft lip and palate, and polydactyly. Her doctor has referred her for genetic counseling to discuss implications for future pregnancies. Which of the following additional information will be MOST helpful to determine the recurrence risk?
a. physical examination on the child’s siblings
b. karyotype on the woman and her partner
c. karyotype results on the child
d. methylation studies on the woman

D. Request an IRB waiver of consent to share de-identified data.
A genetic counselor is the research coordinator of a large study that collects DNA samples from families. The database includes clinical information, family history, and mutation status. A group of researchers conducting a similar study requests access to the research information; however, no mention of sharing information was included in the consent form. Which of the following actions is needed before releasing the information in the database?

A Re-contact and re-consent the population for the new study.
B. De-identify the database information before releasing it.
C. No other action is needed before releasing the information.
D. Request an IRB waiver of consent to share de-identified data.

d. interject the information and prompt the student to continue.
A genetic counselor is supervising a second-year genetic counseling student in a pediatric clinic. During the appointment, the student leaves out an important piece of information. In a positive co-counseling environment, the MOST appropriate response by the supervisor is to
a. provide feedback to the student at the time of the mistake.
b. discuss the missed information with the student after the appointment.
c. interrupt the student and assume counseling responsibilities.
d. interject the information and prompt the student to continue.

C. the plan for the session and if the patient has other concerns to discuss
When contracting with a patient at the beginning of a genetic counseling session, which of the following should the genetic counselor address FIRST?

the patient’s educational background to determine how much information to A.
provide
B. the patient’s affect to determine if there are pressing psychosocial issues
C. the plan for the session and if the patient has other concerns to discuss
D.the patient’s risk factors to determine if other referrals are appropriate

B. “Does he look different from other family members?”
5 yo boy with intellectual disability is referred for genetic evaluation. Family history reveals that his mother has a 30-year-old maternal uncle whom she describes as “slow.” Which of the following questions about the uncle should the genetic counselor ask FIRST to help assess the relevance of this history?

A. “Where does he work?”
B. “Does he look different from other family members?”
“Does he have any children?”
“How old was he when he said his first words?”

A.5-digit zip code
Under the HIPAA privacy rule, which of the following is considered protected health information (PHI) for use in a research study database?

A.5-digit zip code
B. year of birth
D. gender
D. age of disease onset

B Acknowledge her anger and ask her to elaborate on her reaction.
A couple comes for preconception genetic counseling because the woman had bilateral retinoblastoma. The husband asks about his wife’s risks for developing the non-ocular cancers associated with this disease. The genetic counselor invites the woman to answer in an attempt to assess her level of understanding and perception of risk. She responds in anger, “Do you mind not talking about this?!” Which of the following is the MOST appropriate response by the counselor?
A. Rephrase the husband’s question, emphasizing the importance of the
information.
B Acknowledge her anger and ask her to elaborate on her reaction.
C. Recognize her anger and address the husband’s question.
D. Reframe the woman’s strong reaction as an understandable though
inappropriate coping mechanism.

B. MSH2
A 44-year-old man with colon cancer comes for genetic counseling. He has no family history of cancer. His pathology report indicates that his tumor is MSIHigh. During the pre-authorization process, the genetic counselor learns that the patient will have to pay for follow-up molecular testing out of pocket and can afford testing for only two genes. In addition to MLH1, which of the following genes should be tested to identify the causative mutation?
A. EPCAM
B. MSH2
C. MSH6
D. PMS2

C. Her head circumference is above the 98th percentile.
A 55-year-old woman with thyroid cancer is referred for genetic counseling. Her medical history includes surgical excision of a growth on her neck and excision of colon polyps at age 50. She was adopted, and her family history is not available. Which of the following would be the MOST compelling additional information to support testing for mutations in the PTEN gene?
A. Her first son was born at 36 weeks gestation.
B. The growth excised from her neck was a sarcoma.
C. Her head circumference is above the 98th percentile.
D.The polyps were hamartomas.

A. cystic renal disease
(Meckel-Gruber triad: central nervous system malformations (in particular occipital encephalocele), cystic dysplasia of the kidneys, and ductal plate malformation of the liver)
The diagnosis of Meckel-Gruber syndrome is suspected in a stillborn fetus with multiple anomalies. Which of the following features is MOST significant in establishing this diagnosis?

A. cystic renal disease
B. renal agenesis
C. cobblestone lissencephaly
D. postaxial polydactyly

a. “To accurately determine your daughter’s risk for cancer, you must have genetic testing first.”
A 38-year-old woman with breast cancer comes for genetic counseling. Her mother died of ovarian cancer at age 35. The woman states that she is fearful her 19-year-old daughter will develop cancer and wants to know what testing options are available. Which of the following is the genetic counselor’s BEST response?

a. “To accurately determine your daughter’s risk for cancer, you must have genetic testing first.”
b. “To confirm your mother’s diagnosis, we need a copy of her pathology
report.”
c. “It’s understandable to worry about your daughter; we can test her because she is an adult.”
d. “My daughter is also at risk for a BRCA2 mutation, and I worry about her

D. Pendred syndrome (inner ear abnormalities, hearing loss, balance issues, enlargement of vestibular aqueducts)
A 6-year-old boy is referred for genetic counseling. His medical record shows that he had mild congenital hearing loss, which worsened after he fell and hit his head. He has no dysmorphic features and a negative family history. A head CT scan performed after the fall noted bilateral enlargement of the vestibular aqueducts. Which of the following is the MOST likely cause?
a. connexin-26 mutations
b. branchio-oto-renal syndrome
c. mitochondrial A1555G mutation
D. Pendred syndrome

A. autonomy
A 35-year-old woman comes for genetic counseling after recently learning her sister has an MSH2 mutation. The woman wants testing for the familial mutation for herself and her three school-age children. Which of the following issues is MOST important to discuss regarding testing the woman’s children?

A. autonomy
B. beneficence
C. nonmaleficence
D. justice

C. Ask her to describe her understanding of congenital DM.
A 17-year-old pregnant woman and her partner come for genetic counseling at 10 weeks gestation because she has a clinical diagnosis of myotonic dystrophy (DM). She has not had genetic testing. The patient had difficulty in school and dropped out after ninth grade. After reviewing the range of severity of DM, the genetic counselor offers her genetic testing to determine her chance of having a child with congenital DM, which she declines. She states that she does not like needles and would not mind having a child who is like her. Which of the following is the BEST response by the counselor to the woman’s decision to decline testing?
A. Explore the partner’s feelings about the woman having genetic testing for
DM.
B. Support her decision to decline testing, and conclude the session.
C. Ask her to describe her understanding of congenital DM.
D.Encourage her not to base her decision for testing on a fear of needles.

a. Reassure her that the risk for FAS is not increased given the timing of the exposure.

  1. A 32-year-old pregnant woman is referred for genetic counseling at 6 weeks gestation because of alcohol exposure. She explains that she drank several alcoholic beverages daily for one week during her honeymoon 4 weeks ago. She
    expresses concern that this exposure will cause fetal alcohol syndrome (FAS). Which of the following is the BEST response to this patient?
    a. Reassure her that the risk for FAS is not increased given the timing of the exposure.
    b. Reassure her that the risk for FAS is not increased given the limited
    duration of the exposure.
    C. Confirm that there is an increased risk for FAS since there is no known safe amount of alcohol consumption during pregnancy.
    D. Confirm that there is an increased risk for FAS and probably other birth
    defects because organogenesis has begun.

B. retinitis pigmentosa
Usher syndrome is an autosomal recessive disorder that includes deafness and which of the following?

A. anosmia
B. retinitis pigmentosa
C. goiter
D. kidney disease

A. 2%.
A 50-year-old woman tests negative for a familial mutation in the BRCA1 gene. She plans to discuss ovary removal with her gynecologist because she believes her risks are still high. The genetic counselor should clarify that the woman’s lifetime risk of ovarian cancer is closest to

A. 2%.
B. 5%.
C. 10%.
D.40%.

B. high free beta hCG, low PAPP-A, nuchal translucency greater than 3 mm
Which of the following first-trimester screening patterns is most suggestive of trisomy 21?
high free beta hCG, high PAPP-A, nuchal translucency greater than 3 mm
B. high free beta hCG, low PAPP-A, nuchal translucency greater than 3 mm
C. low free beta hCG, high PAPP-A, nuchal translucency less than 3 mm
D. low free beta hCG, low PAPP-A, nuchal translucency less than 3 mm

A. steroid sulfatase deficiency
A 25-year-old pregnant woman comes for genetic counseling. Her screening results are positive for an increased risk for Smith-Lemli-Opitz (SLO) syndrome. Her results are as follows:
AFP: 1 MoM
HcG: 0.95 MoM
Ue3: 0.1 MoM
Inhibin: 1.1 MoM
In addition to counseling the woman about SLO, which of the following should be discussed?

A. steroid sulfatase deficiency
trisomy 21
biotinidase deficiency
trisomy 18

A. 0%, mitochondrial inheritence
28-year-old pregnant woman comes for genetic counseling because the father of her female fetus has a diagnosis of NARP. The risk that this fetus has inherited this condition is closest to which of the following percentages?

A. 0%
B. 25%
C. 50%
D. 100%

B. 1%
A couple comes for genetic counseling because their newborn son has an abnormal karyotype
(46XY, i(21)(q10)). The father has an unaffected son, the mother has an unaffected daughter). The woman is 39 years old and her husband is 41 years old. Which of the following is closest to the recurrence risk for the condition?

A. 0%
B. 1%
C. 50%
D. 100%

C. Gaucher disease
A 35-year-old Jewish woman of Russian and Polish ancestry is referred for genetic counseling for ethnic-based carrier testing. She is MOST likely to be a carrier of which of the following conditions?

A. Canavan disease
B. cystic fibrosis
C. Gaucher disease
D.Tay-Sachs disease

C. chromosomal microarray
A 5-year-old girl is referred to the genetics clinic for evaluation of dysmorphic features and developmental delay. She has fair skin, microcephaly, a prominent jaw, and a wide-based gait. She has had seizures since she was 6 months old, walked at 23 months, and has no speech. Family history is non-
contributory. Which of the following evaluations is MOST likely to establish the diagnosis?

A. FMR1 testing
B. MECP2 gene sequencing
C. chromosomal microarray
D.urine organic acid screen

D.Velocardiofacial syndrome
Which of the following conditions warrants patient resource materials written to a lower than average literacy level?
A.Stickler syndrome
B. Maroteaux-Lamy syndrome
C. Von Hippel Lindau syndrome
D.Velocardiofacial syndrome

B.Inquire about how the patient has made difficult decisions in the past.
A 38-year-old woman comes for genetic counseling because of an abnormal first trimester screen with an increased risk for fetal Down syndrome. She listens to the genetic counselor explain the additional prenatal testing and screening options, and at the end tells the counselor, “I don’t know what I should do.” Which of the following is the BEST way for the counselor to approach this situation?

A. Restate the testing and screening options in simpler terms.
B.Inquire about how the patient has made difficult decisions in the past.
C. Provide the patient with balanced resources that cover all available options.
D. Discuss the patient’s personal support systems and who might help her
reach a decision.

D. OTIS (Organization of Teratology Information Specialists)
40-year-old pregnant woman comes for genetic counseling. She tells the genetic counselor that she has been taking paroxetine for an anxiety disorder and is concerned about the effects of this drug on the fetus. Which of the following resources is BEST to determine the risk to the fetus?

A. GeneTests
B. OMIM
C. March of Dimes
D. OTIS

C. escape-avoidance
A 40-year-old woman with Li-Fraumeni syndrome comes for genetic counseling after testing confirms that she has a TP53 mutation. Her asymptomatic 18-year old son is also found to have the same mutation. The mother repeatedly suggests that gene therapy should be available soon to repair the gene for her son and that perhaps his test should be repeated in case there was a mix-up at the laboratory. The mother is exhibiting which of the following coping styles?

A. confrontive
B. distancing
C. escape-avoidance
D.positive reappraisal

D. Recommend CFTR deletion/duplication testing.
A couple returns for genetic counseling to discuss their genetic test results for cystic fibrosis (CF). Their daughter had an abnormal sweat test and CFTR sequencing that identified one known mutation and one variant of unknown significance (VUS). Parental testing revealed that both the mutation and VUS were maternally inherited. Which of the following should the genetic counselor do next?
A. Tell the family their daughter is a carrier and no further testing is warranted.
B. Tell the family that more research is needed to determine the significance of the VUS.
C. Recommend paternal uniparental disomy testing.
D. Recommend CFTR deletion/duplication testing.

A.chromosome deletion (Prader-Willi: hypotonia ass. w/ poor suck, genital hypoplasia, sleep disorders)
A 2-month-old boy with failure to thrive is seen in the genetics clinic for evaluation and counseling. He has poor suck, and his parents have to wake him to feed during the night. He has hypotonia and genital hypoplasia. Which of the following mechanisms is the MOST likely cause of this condition?

A.chromosome deletion
b. uniparental disomy
c. imprinting mutation
c. trinucleotide repeat

a. “Tell me more about how you have been feeling before and during the pregnancy.”
A woman is seen for prenatal genetic counseling due to an abnormal maternal serum screen. The counselor notes that the patient is frequently tearful and seems to have trouble concentrating. The patient says that she, “Just can’t sleep.” Which of the following is the BEST initial response from the counselor?

a. “Tell me more about how you have been feeling before and during the pregnancy.”
b. “Would you consider meeting with a psychiatrist for evaluation of your
sadness?”
c. “It is normal to feel down or even depressed after hearing about this kind of
result.”
d. “Who have you turned to for emotional support since hearing about the test
result?”

D.Reschedule the appointment when a professional interpreter is available.

  1. A 41-year-old pregnant Vietnamese woman at 15 weeks gestation comes for genetic counseling prior to amniocentesis. She speaks very little English and has brought her husband with her to act as an interpreter. The hospital’s Vietnamese interpreter is not currently available. Which of the following is the BEST action by the genetic counselor?

Call the patient’s obstetrician for advice on how to proceed.
Conduct the session with the husband translating.
Proceed with the amniocentesis with counseling afterwards.
D.Reschedule the appointment when a professional interpreter is available.

B.intellectualization
During a genetic counseling session, a 27-year-old woman is informed that she has a BRCA1 gene mutation. The woman does not seem distressed. She asks detailed questions about the molecular nature of the mutation, the cancer risks associated with the BRCA1 mutation, and the medical management options. She uses correct genetic and medical terminology such as “autosomal dominant Mendelian inheritance,” “protein truncation,” and “stop codon.” This patient appears to be coping with the genetic test results using which of the following defense mechanisms?

A. compartmentalization
B.intellectualization
C. dissociation
R. rationalization

D. “How has this situation been isolating for you?”
A woman comes for genetic counseling to discuss the prenatal diagnosis of trisomy 21 by amniocentesis. She says that she feels “very alone” with this diagnosis. Which of the following is the genetic counselor’s BEST initial response?

“Who has been supporting you during the pregnancy?”
“Have you spoken with a trisomy 21 support group?”
“What has your partner said about the diagnosis?”
D. “How has this situation been isolating for you?”

B. 1/150.
A non-consanguineous Caucasian couple is referred for genetic counseling. The woman’s brother died of cystic fibrosis (CF). The risk for the couple to have a child with CF is closest to

A. 1/100.
B. 1/150.
C. 1/250.
D. 1/400.

C. Parkinson disease. (FXTAS)
A 50-year-old man has 120 CGG repeats in the FMR1 gene. He is at risk to develop symptoms that could be mistaken for
A. frontotemporal dementia.
B. Huntington disease.
C. Parkinson disease.
D.spinocerebellar ataxia.

a. “What medications were you prescribed, and when did you stop taking them?”
A newborn male with hydrocephalus, microtia, and a ventricular septal defect is referred to the genetics clinic. His mother had been treated for cystic acne, but she reports that she stopped taking all medicines when she learned she was pregnant. When obtaining the pregnancy history, which of the following is MOST important to ask?

a. “What medications were you prescribed, and when did you stop taking them?”
b. “What medications were you prescribed, and how long did you take them
prior to conception?”
c. “Did you have to go on a special diet prior to pregnancy?”
d. “Did you have fever or flu-like symptoms in early pregnancy?”

c. countertransference
A 28-year-old man with Klinefelter syndrome is referred for genetic counseling. He calls the genetic counselor several times with questions. The man’s voice and personality are similar to the counselor’s brother. She finds it easy to talk to him and enjoys his questions. She realizes her calls with him are taking longer than they should. Which of the following BEST describes this situation?

a. ambivalence
b. projection
c. countertransference
d.transference

Line connecting two female partners, line connecting mother to pregnancy, line connecting male w/ D to pregnancy
A 38-year-old pregnant woman is referred during her first trimester for genetic counseling. Her female partner accompanies her to this visit. The pregnancy was conceived using donor sperm. How is this reproductive scenario indicated when drawing the pedigree?

B. stomach cancer (maybe because of CDH1? and stomach cancer could be colon, ovarian, uterine, etc)
A 35-year-old healthy woman comes for genetic counseling for cancer risk assessment. Her mother died with breast cancer at age 43. Two maternal aunts and one maternal uncle are all cancer-free. Her maternal grandmother had stomach cancer in her 60s. The consultant’s father is healthy at 55. The father’s siblings are cancer-free, but his brother’s daughter was diagnosed with cervical cancer in her 30s. The consultant’s paternal grandfather had colon cancer in his 60s. Pathologic confirmation of which reported cancer will have the largest impact on the risk assessment for this consultant?

A. breast cancer
B. stomach cancer
C. cervical cancer
D. colon cancer

C. Forward her request to the compliance officer.
A woman who is seen for genetic counseling contacts the genetic counselor after the session. She states that she wants her history of two prior elective abortions removed from her medical record. After acknowledging her request, which of the following is the BEST response by the counselor?

A. Alter the record as requested.
B. Tell her that the medical record cannot be altered.
C. Forward her request to the compliance officer.
D. Request a consultation with the ethics board.

B. McCune-Albright syndrome (cafe au lait, bone deformities, precocious puberty)
A 3-year-old girl is referred for a genetic evaluation. Her medical record states that she has several large café au lait spots, multiple long-bone fractures, and recent onset of menstrual bleeding. Which of the following is the MOST likely diagnosis?

A. Noonan syndrome
B. McCune-Albright syndrome
C. neurofibromatosis
D. osteogenesis imperfecta

D. diabetes (caudal regression aka sacral agenesis associated with T1D/T2D not gestational diabetes)
A 33-year-old woman who is 20 weeks pregnant is referred for genetic counseling. Fetal ultrasound showed caudal regression. Which of the following conditions is the woman MOST likely to have?

a. lupus
b. phenylketonuria
c. hypothyroidism
D. diabetes

D. Explain that she and all of her children are at risk. (SCAiii is AD, onsets later in life)
A 25-year-old woman comes for genetic counseling due to her family history of SCA type III. She is interested in preimplantation genetic diagnosis (PGD). She states that she will only have female embryos implanted because she does not want any of her children to have the condition. Which of the following is the BEST way to counsel the patient?

A. Explain that none of her children will be affected.
B. Agree that only her male children are at risk but explain that PGD is not
offered for adult onset conditions.
C. Agree with her that only her male children are at risk and help arrange PGD.
D. Explain that she and all of her children are at risk.

C. Report the information to child protective services or the local police department.
A 38-year-old woman comes for genetic counseling because of advanced maternal age. The genetic counselor asks the patient about a cast on her arm. She states that her husband threw a chair at her in a fit of rage. The genetic counselor explores this further. The patient tells her that her husband has been abusive in the past, and she is afraid for their children. The patient believes the injury was her fault since she provoked her husband’s anger. The counselor suggests that the patient speak with a social worker, but the patient declines. Which of the following is the MOST appropriate action by the counselor?

A. Ask the patient for permission to speak to the husband about the incident.
B. Encourage the patient to seek help about her abusive relationship.
C. Report the information to child protective services or the local police department.
D. Respect the patient’s wishes and keep this information confidential.

c. “I can arrange for you to talk with people in a similar situation.”
The parents of a child newly diagnosed with complete androgen insensitivity
come for genetic counseling. The genetic counselor tells the parents it is common practice to remove the streak gonads. The mother asks, “Do you think that we should agree to this?” Which of the following is the BEST response by the counselor?

a. “Genetic counselors can’t advise you one way or the other.”
b. “It doesn’t matter what I think; I am not in your shoes.”
c. “I can arrange for you to talk with people in a similar situation.”
d. “Surgery is necessary to reduce the chance of cancer.”

B. dystrophin deletion/duplication analysis followed by sequencing (deletions/duplications most common)
A 4-year-old boy is referred to a genetics clinic because he has calf hypertrophy and trouble rising from the floor when seated. His creatine kinase level is 10,000 U/L. Which of the following is the BEST initial method to determine his diagnosis?

A. dystrophin immunohistochemistry on muscle tissue
B. dystrophin deletion/duplication analysis followed by sequencing
C. Western blot analysis for dystrophin on muscle tissue
D. FISH analysis for deletion/duplication detection

B.3-5%.
A couple seeks counseling because their first child was born with a ventricular septal defect. There is no family history of any congenital heart disease, and the parents have normal cardiac examinations. They should be counseled that the risk that a subsequent child will have a similar or other cardiac defect is closest to

A. less than 1%.
B.3-5%.
C.8-10%.
D.25%.

A. comprehensive germline large rearrangement analysis of BRCA1 and
BRCA2
A 60-year-old woman who was diagnosed with breast cancer at age 36 and ovarian cancer at age 50 is being seen for genetic counseling. Her mother was diagnosed with breast cancer at age 55; a maternal aunt was diagnosed with ovarian cancer at age 60; and a maternal first cousin was diagnosed with leukemia at age 20. The proband underwent comprehensive germline sequence analysis of BRCA1 and BRCA2 at the time of her ovarian cancer diagnosis with negative results. Which of the following additional genetic tests would be MOST appropriate for the proband to have?
A. comprehensive germline large rearrangement analysis of BRCA1 and
BRCA2
B. BRCA1 and BRCA2 promoter methylation analysis of either of her tumors
C. comprehensive germline PTEN genetic testing
D. P53 promoter methylation analysis of either of her tumors

a. Ask the father about his motivation for requesting genetic testing of his daughters.
A man comes for genetic counseling with his two daughters, ages 12 and

  1. Their mother died of breast cancer at age 39 and had a BRCA1 mutation. The father wants his daughters to be tested for the mutation. Which of the following is the BEST next step by the genetic counselor?

a. Ask the father about his motivation for requesting genetic testing of his daughters.
b. Schedule a genetic counseling appointment for the daughters to evaluate
their opinions and feelings about testing.
c. Explain the life insurance implications of a positive result.
d. Arrange genetic testing for the 17-year-old daughter.

D.malformation.
A woman who is 27 weeks pregnant has a fetal ultrasound that is suggestive of esophageal atresia. This anomaly is an example of

A.disruption.
B.deformation.
C.sequence.
D.malformation.

D.isovaleric acidemia (sweaty feet smell during crisis is main characteristic)
A 25-year-old woman comes for genetic counseling at 12 weeks gestation because her sister recently had a baby girl with a “metabolic condition.” She was told that her niece has recurrent vomiting, seizures, and smells like “sweaty feet.” The niece was born overseas, medical records cannot be obtained, and the woman is estranged from her sister. The genetic counselor should look for laboratories that offer carrier testing for which of the following conditions?

A. maple syrup urine disease
B. propionic acidemia
C. methylmalonic acidemia
D.isovaleric acidemia

A. consultant
When a supervisor and his student work collaboratively on a project with mutually agreed-upon objectives and the focus is on the student’s clients, which of the following roles is the supervisor using?

A. consultant
B. teacher
C. counselor
D. evaluator

B. explain the importance of obtaining family history information.
A couple schedules an appointment to bring their 5-year-old son with autism to the genetics clinic to discuss genetic testing. The genetic counselor calls prior to the visit to obtain a family history. The couple is reluctant to discuss details and asks why this information is needed, since they only want to do testing. The BEST next step would be for the counselor to:

A. provide recommendations for genetic testing to the primary care provider
and schedule a results appointment
B. explain the importance of obtaining family history information.
C. refuse to schedule an appointment given that the family is not cooperating.
D. schedule an appointment for genetic counseling to further discuss family
history concerns.

C. hemochromatosis (serum ferritin = iron in blood)
A 40-year-old man is referred to the genetics clinic by his internist. He has chronic fatigue, joint pain in his hands and neck, and type 2 diabetes. Complete blood count, thyroid function studies, and lipid profiles are within the normal ranges, but serum ferritin levels are increased. Which of the following is the MOST likely diagnosis?

A. acute intermittent porphyria
B. Fabry disease
C. hemochromatosis
D.Wilson disease

B. the age at which the uncle began having symptoms of HD
A 30-year-old man is scheduled for genetic counseling because his maternal uncle recently died with Huntington disease (HD). The man’s mother died in a car accident at age 25. Which of the following should the genetic counselor ask to BEST assess the patient’s risk of developing HD?
A. if any of the uncle’s children have developed HD
B. the age at which the uncle began having symptoms of HD
C. the severity of the uncle’s HD symptoms
D. if the patient has experienced episodes of memory loss

A.hemoglobin electrophoresis and iron studies on the woman
A 38-year-old African American woman comes for genetic counseling at 10 weeks gestation. She is not accompanied by her partner. She has a low mean corpuscular volume. Which of the following is the BEST next step for the genetic counselor to recommend?

A.hemoglobin electrophoresis and iron studies on the woman
B. genetic testing for conditions associated with low MCV for the partner
C. complete blood count on the partner
D. genetic testing for conditions associated with low MCV for the woman

C. 1/5
The father and paternal grandfather of a 44-year-old asymptomatic woman both died of Huntington disease (HD). Approximately 75% of people who have the expanded allele for HD show clinical signs of the disease by age 44. What is the woman’s risk for HD?

A. 1/2
B. 1/4
C. 1/5
D. 1/10

A. Consult with the ethics review board regarding the best course of action.
A 25-year-old pregnant woman comes for genetic counseling and amniocentesis because her father had Becker muscular dystrophy (BMD). Ultrasonography shows a male fetus. Her father’s mutation is known. Test results show that neither the woman nor her fetus carries the BMD mutation. The woman is not aware that her father is not her biological father. Which of the following is the BEST course of action for the genetic counselor?

A. Consult with the ethics review board regarding the best course of action.
B. Tell the woman that the results indicated that her fetus is not affected.
C. Arrange a session for the woman and her mother to discuss nonpaternity.
D. Contact the woman’s obstetrician with the test results, and let him inform
her.

C. incontinentia pigmenti syndrome. (whorls and tooth/nail abnormalities)
A 3-year-old girl comes for genetic evaluation because of developmental delay and irregular brownish whorls on her left leg. Her mother reports that the skin discoloration was first noted at about 2 months of age and that tooth eruption was delayed. The MOST likely diagnosis is

A. neurofibromatosis.
B. linear sebaceous nevus syndrome.
C. incontinentia pigmenti syndrome.
D.tuberous sclerosis.

B. 20%
A woman comes for genetic counseling because of a family history of “muscle disease.” She has two teenage sons who appear healthy. Her 48-year-old maternal uncle is barely ambulatory. Her sister’s 17-year-old son has muscle weakness and a serum creatine kinase level greater than 5000 U/L. No other family members are known to be affected. The woman’s two sons have creatine kinase testing, and they both have levels less than 150 U/L. What is the woman’s risk to be a carrier for the condition in her family?

A. 10%
B. 20%
C. 40%
D. 50%

C. Discuss her emotional response.
A 37-year-old woman is referred to the adult genetics clinic to be evaluated for Ehlers-Danlos Syndrome (EDS) because she has a personal history of hypermobile joints, fibromyalgia, and inflammatory bowel disease. When the counselor is taking the family history, the woman says, “All these questions aren’t important! What does this have to do with me?” Which of the following is the genetic counselor’s BEST next step?

A. Redirect to the goals of the session.
B. Review the importance of obtaining the family history.
C. Discuss her emotional response.
D.Acknowledge that patients with EDS are usually anxious.

B. CDKN2A
A 45-year-old man is referred for genetic counseling because he has pancreatic cancer. Review of his medical record indicates that he also had a melanoma. He is MOST likely to have a mutation in which of the following genes?
A. BRCA1
B. CDKN2A
C. PTEN
D. TSC2

B. Ask the couple what they understand about their child’s condition.
A couple comes for genetic counseling because their child has a rare chromosome abnormality. Which of the following is the BEST first step?

A. Provide a review of the condition while checking in with the couple.
B. Ask the couple what they understand about their child’s condition.
C. Reflect that the couple is likely having a difficult time with this diagnosis.
D. Outline the topics of the genetic counseling session.

A. ophthalmologic evaluation
A 12-year-old girl is referred for a genetic evaluation to rule out a syndrome after having been diagnosed with colon polyps. The abnormal results of which of the following would be MOST supportive of a diagnosis of familial adenomatous polyposis?
A. ophthalmologic evaluation
B. audiologic testing
C. brain MRI
D. abdominal ultrasound

B. 45,XX,t(13;15)(q10;q10) (a is turners, c is unbalanced translocation, d is robertsonian t21)
Which of the following karyotypes is MOST consistent with a normal phenotype?

A. 45,X
B. 45,XX,t(13;15)(q10;q10)
C. 46,X,der(X),t(X;5)(p31;p21)
D. 46,XX,t(14;21)(q10;q10)+21

D. no treatment
A couple comes for genetic counseling because their daughter has congenital adrenal hyperplasia (CAH). Which of the following should be recommended for future pregnancies?

A. BH4 supplementation
B. biotin therapy
C. cholesterol supplementation
D. no treatment

A. cloverleaf skull (thanatophoric dysplasia = “death bearing” short limbs, extra skin folds, type II has cloverleaf skull, either stillborn or infant death )
A 28-year-old female is seen by a genetic counselor during her second pregnancy because she reports that her first pregnancy was diagnosed with thanatophoric dysplasia. No records are available to confirm this. In order to substantiate the diagnosis, it is BEST to ask whether the fetus had which of the following in addition to shortened limbs?

A. cloverleaf skull
B. fractures
C. ambiguous genitalia
D. polydactyly

C. refer the family to the study coordinator.
All testing has been negative in a 5-year-old female with multiple congenital anomalies and intellectual disability. The child’s parents are interested in enrolling their daughter in a research study to investigate potential genetic causes. The consent forms are available from the research study’s website. The BEST next step to facilitate enrollment of this child into the research study is to

A. obtain informed consent using the online form.
B. obtain assent from the 5-year-old daughter.
C. refer the family to the study coordinator.
D. draw a blood sample for the research study.

D. reciprocity.
A Caucasian genetic counselor meets with a Hispanic woman who is being evaluated for familial adenomatous polyposis. In an attempt to gain the client’s trust and to promote their engagement, the counselor begins the conversation by
sharing information about her own family’s experience with cancer. This counseling technique is based on

A. attending language.
B. congruence.
C. shared language.
D. reciprocity.

A, Bardet-Biedl syndrome (Meckel = kidney cysts/encephalocele, Pallister-Hall = polydactyly and hypothalamic hamartoma, Smith-Lemli-Opitz = microcephaly, ID, multiple birth defects)
While a genetic counselor is taking a family history, the patient reports that of her sister’s four children, two have polydactyly, cognitive impairment, obesity, and vision loss. Which of the following syndromes is the MOST likely explanation of their features?

A. Bardet-Biedl syndrome
B. Meckel-Gruber syndrome
C. Pallister-Hall syndrome
D. Smith-Lemli-Opitz syndrome

AD Advanced paternal age conditions
Achondroplasia
Neurofibromatosis
Marfan
Treacher-collins
Waardenburg
Thanophoric dysplasia
OI
Apert

XLR Advanced paternal age conditions
Fragile x
Hemophilia A and B
DMD
Incognita pigmenti
Hunter
RP

Leave a Comment

Scroll to Top